Saturday, December 31, 2016

www.CBSEPORTAL.COM - : (Download) CBSE Class-10 2016-17 Sample Paper And Marking Scheme (Hindi-A)

www.CBSEPORTAL.COM - : (Download) CBSE Class-10 2016-17 Sample Paper And Marking Scheme (Hindi-A)

Link to CBSE PORTAL : CBSE, ICSE, NIOS, JEE-MAIN, AIPMT Students Community

(Download) CBSE Class-10 2016-17 Sample Paper And Marking Scheme (Hindi-A)

Posted: 30 Dec 2016 04:38 AM PST

(Download) CBSE Class-10 2016-17 Sample Paper And Marking Scheme (Foundation of Information Technology)

Posted: 30 Dec 2016 04:17 AM PST

(Download) CBSE Class-10 2016-17 Sample Paper And Marking Scheme (Foundation of Information Technology)

Max Time: 3

hours Max Marks: 70

SECTION A

Q1) Fill in the blanks: [5]

i) Viruses, worms and trojans are collectively named ________________.

ii) Cellspacing attribute is used with ______________ tag.

iii) _____________ is a program which makes you view the information on the World Wide Web.

iv) Collection of Web Pages is called _____________.

v) Columns of a table are specified using __________tag.

vi) _________ can replicate itself automatically .

vii)_________ attribute is used with <A> tag to specify the URL of link.

viii) ___________ is type of program that is installed on a user's computer to collect information about user.

ix) XML attribute value should always be enclosed in _______________ .

x) ______________is a program that appears harmless but actually performs malicious functions.

Q2) State True or False: [5]

i) The World Wide Web Consortium was founded to develop common standards.

ii) An XML document can have any number of child elements.

iii) No value is specified with border attribute of <img> tag.

iv) COLSPAN attribute of <TD> tag is used to merge more than one column of table.

v) Unsolicited bulk emails are called spam.

vi) Malware is Antivirus software.

vii) Because each computer differs in terms of what fonts it can display, each individual browser determines how text is to be displayed.

viii) Xml is Extended MarkupLanguage

ix) Software piracy is crime.

x) A cracker is a malicious programmer who break into secure systems.

Q3) Short Answer Questions –

i. What does an anti-virus software do? [2]

ii. What is full form of HTTP and what is its use? [2]

iii. Write the HTML code to display links of a web page in yellow colour. [2]

iv. Write the full forms of: [2]
i) URL
ii) <LI>
iii) <TD>
iv) HTML

v. As life gets busier, it becomes difficult for everyone to keep a track with school & college friends, old colleagues, old neighbours and favourite teachers. It is important to keep in touch with all your near and dear ones. At times, people sitting miles away doing similar kind of activity or solving similar kind of problems can help you to achieve goals faster by sharing their experiences. Similarly people belonging to different socio-economic background can change your perspective and can enhance your understanding of various cultures. [2]
a. Suggest two real time tools that are suitable for the above-mentioned activities.
b. What is the generic name used for such tools?

vi. Laleema Chakradhar wants a broadband connection for accessing her mails and staying informed about the latest happenings in the field of Biotechnology. Can you suggest two Internet Service Providers of India to be approached for the same? [2]

Q4) Short Answer Questions –
i. Draw the XML tree for the code given below: 

Click Here To Download Full Sample Paper

Click Here To Download Full Marking Scheme

<<Go Back To Main Page    

(Download) CBSE Class-12 2016-17 Sample Paper And Marking Scheme (Engineering Graphics)

Posted: 30 Dec 2016 03:35 AM PST

(Download) CBSE Class-12 2016-17 Sample Paper And Marking Scheme (Engineering Graphics)
 

Time allowed: 3 hours

Maximum marks: 70

Instructions:

i. Attempt all the questions.
ii. Use both sides of the drawing sheet, if necessary.
iii. All dimensions are in millimetres.
iv. Missing and mismatching dimensions, if any, may be suitably assumed.
v. Follow the SP: 46-2003 revised codes (with first angle  method of projection).
vi. In no view of question2, are hidden edges or lines required.
vii. In question 4, hidden edges or lines are to be shown in views without section.
viii. Number your answers according to questions.

Q.1 Answer the following Multiple Choice Questions. Print the correct choice on your drawing sheet 5 X 1=5

(i) What is the angle in degree between the main scale and isometric scale  in the construction of isometric scale?

a) 300
b) 150
c) 45o
d) 90o

(ii) Which is the modified form of square thread?

a) V-thread
b) Metric thread
c) Knuckle thread
d) B.S.W thread

(iii) What is the width of the Rectangular Sunk Key, if the diameter of the shaft is D?

a) D/2
b) D/8
c) D/4
d) D/6

(iv) Name the joint used for joining two pipes

a) Flanged pipe joint
b) Bushed bearing
c) Turn buckle
d) Cotter joint

(v)Name the portion between the rim and the hub of a cast iron pulley

a) Shaft
b) Gap
c) Bush
d) Web

Q.2 (i) Construct an isometric scale. 4

(ii) A frustum of a pentagonal pyramid (base side 50mm, top side 30mm and axis 70mm) is kept with its axis perpendicular to H.P. One of its base sides parallel to V.P. and away from it. Draw its isometric projection. Show the  axis and indicate the direction of viewing .Give all dimensions. 7

(iii) A hexagonal pyramid (base edge 30mm, axis 50mm), having two of its hexagonal edges parallel to V.P., is placed centrally on the top square face of a square slab(base side 80mm, height 20mm). The common axis is perpendicular to H.P. Draw the isometric projection of the combination of solids. Show the common axis and indicate the direction of viewing. Give all dimensions. 13

Q.3. (i) Draw to scale 1:1 the front view and top view of a hexagonal nut, with vertical axis (diameter 20mm). Give standard dimensions. 8

OR

Draw to scale 1:1, the front view and side view of a Square headed bolt of diameter 20mm, keeping the axis parallel to V.P and H.P. Give standard dimensions.

(ii) Sketch freehand the front view and top view of a Grub screw of size M25, Keeping the axis vertical. Give standard dimensions. 5

OR

Sketch freehand the front view and side view of a Plain stud, of diameter 20mm, keeping the axis horizontal. Give standard dimensions.

Q.4 Figure 1 shows the details of the parts of a FLANGE PIPE JOINT. Assemble these parts correctly, and then draw the following views using scale 1:1.

(i) Front view, top half in section. 14
(ii) Left side view. 8

Print the title and the scale used. Draw the projection symbol. Give 6 important dimensions.

 

Click Here To Download Full Sample Paper

Click Here To Download Full Marking Scheme

<<Go Back To Main Page  

(Download) CBSE Class-12 2016-17 Sample Paper And Marking Scheme (Economics)

Posted: 30 Dec 2016 03:11 AM PST

(Download) CBSE Class-12 2016-17 Sample Paper And Marking Scheme (Economics)

Time : 3 Hours Maximum Marks : 100

Instructions:

1. All questions in both sections are compulsory. However, there is internal choice in some questions.
2. Marks for questions are indicated against each question.
3. Question No.1-5 and 16-20 are very short answer questions carrying 1 mark each. They are required to be answered in one sentence.
4. Question No.6-8 and 21-23 are short answer questions carrying 3 marks each. Answers to them should not normally exceed 60 words each.
5. Question No.9-11 and 24-26 are also short answer questions carrying 4 marks each.
Answers to them should not normally exceed 70 words each.
6. Question No.12-15 and 27-30 are long answer questions carrying 6 marks each. Answers to them should not normally exceed 100 words each
7. Answers should be brief and to the point and the above word limit be adhered to as far as possible.

Section A: Microeconomics

1. When is a consumer said to be rational? (1)

2. Define normative economics, with a suitable example. (1)

3. State the meaning of `quantity demanded of a commodity`. (1)

4. If a firm's production department data says that the total variable cost for producing 8 units and 10 units of output is 2,500 and 3,000 respectively, marginal cost of 10th unit will be
a. 100
b. 150
c. 500
d. 250

5. State any one assumption for the construction of the curve that shows the possibilities of potential production of two goods in an economy. (1)

6. State the behavior of Marginal Physical Product, under Returns to a Factor. (3)

7. Using appropriate schedules, briefly describe the determination of market equilibrium. (3)

8. "In a hypothetical market of mobile phones, the brand AWAAZ was leading the market share. Its nearest competitor VAARTA suddenly changed its strategy by bringing in a new model of the mobile phone at a relatively lesser price. In response, AWAAZ too slashed its price."

Based on the above information, identify the form of market represented and discuss any one feature of the market.

Or

Discuss the primary reason for 'indeterminateness of demand curve' under the oligopoly form of market. (3)

9. a. Arrange the following coefficients of price elasticity of demand in ascending order: -0.7, -0.3, -1.1, -0.8
b. Comment upon the degree of elasticity of demand for Good X, using the total outlay method, if the price of X falls from 18 per unit to 13 per unit and its quantity demanded rises from 50 units to 100 units. (1+3)

10. Identify which of the following is not true for the Indifference Curves. Give valid reasons for choice of your answer:
a. Lower indifference curve represents lower level of satisfaction.
b. Two regular convex to origin indifference curves can intersect each other.
c. Indifference curve must be convex to origin at the point of tangency with the budget line at the consumer's equilibrium.
d. Indifference curves are drawn under the ordinal approach to consumer equilibrium.

OR

A consumer has total money income of 250 to be spent on two goods X and Y with prices of 25 and 10 per unit respectively. On the basis of the information given, answer the following questions:
a. Give the equation of the budget line for the consumer.
b. What is the value of slope of the budget line?
c. How many units can the consumer buy if he is to spend all his money income on good X?
d. How does the budget line change if there is a fall in price of good Y? (4)

11. Explain the concept of marginal opportunity cost using a numerical example. (4)

12. Define Price Floor. What is the common purpose of fixation of floor price by the government? Explain any one likely consequence of this nature of intervention by the government.

OR

Define Price Ceiling. What is the common purpose for the price ceiling imposed by the government? Explain any one likely consequence of this nature of intervention by the government in the price determination process. 

 

Click Here To Download Full Sample Paper

Click Here To Download Full Marking Scheme

<<Go Back To Main Page  

NCERT Social Science Question Paper (Class-9)

Posted: 29 Dec 2016 09:39 PM PST

NCERT Social Science Question Paper (Class-9)


(Political Science) Chapter 1 Democracy in the Contemporary World


Question 1: Which of the following does not lead to the spread of democracy?

(a) Struggle by the people
(b) Invasion by foreign countries
(c) End of colonialism
(d) People's desire for freedom

Question 2: Which of the following statements is true about today's world?

(a) Monarchy as a form of government has vanished.
(b) The relationship between different countries has become more democraticever before.
(c) In more and more countries rulers are being elected by the people.
(d) There are no more military dictators in the world.

Question 3: Use one of the following statements to complete the sentence: Democracy in the international organisations requires that …

(a) The rich countries should have a greater say.
(b) Countries should have a say according to their military power.
(c) Countries should be treated with respect in proportion to their population.
(d) All countries in the world should be treated equally.

Question 4: Based on the information given in this chapter, match the following countries anpath democracy has taken in that country.

Question 6: Which freedoms are usually taken away when a democracy is overthrown bmilitary?

Question 7: Which of the following positions can contribute to democracy at the global level? Give reasons for your answer in each case.

(a) My country gives more money to international institutions. Therefore, I want to be treated with more respect and exercise more power.
(b) My country may be small or poor. But my voice must be heard with equal respect, because these decisions will affect my country.
(c) Wealthy nations will have a greater say in international affairs. They cannot let their interests suffer just because they are outnumbered by poor nations.
(d) Big countries like India must have a greater say in international organisations.

Question 8: Here are three opinions heard in a television debate on the struggle for democracy inNepal. Which of these do you agree with and why?

Guest 1: India is a democracy. Therefore, the Indian government must support thpeople of Nepal who are struggling against monarchy and for democracy.
Guest 2: That is a dangerous argument. We would be in the same position as the USwas in Iraq. Remember, no outside force can promote democracy.
Guest 3: But why should we bother about the internal affairs of another countryWe should be worried about our business interests there, not about democracy.

Question 9: In an imaginary country called Happyland, the people overthrew the foreign ruler and brought back the old royal family. They said: "After all their ancestors were our kings before foreigners started ruling us. It is good that we have one strong ruler, who can help us become rich and powerful". When someone talked about democracy the wise men said it is a foreign idea. Their struggle was to throw the foreigners and their ideas out of the country. When someone demanded freedom for the media, the elders thought that too much criticism of the ruler would not help them improve their living standards. "After all, the king is so kind and interested in the welfare of all the subjects. Why create problems for him. Don't we all want to be happy?" After reading the above passage, Chaman, Champa and Chandru made the following observations. Chaman: Happyland is a democratic country because people were able to throw out the foreign rulers and bring back the king. Champa: Happyland is not a democratic country because people cannot criticise the ruler. The king may be nice and may provide economic prosperity, but a king cannot

give a democratic rule. Chandru: What people need is happiness. So they are willing to allow their new ruler to take decisions for them. If people are happy it must be a democracy. What is your opinion about each of these statements? What do you think about the form of government in this country?


(Political Science) Chapter 2 What is Democracy Why Democracy


Question 1: Here is some information about four countries. Based on this information, how woyou classify each of these countries. Write 'democratic', 'undemocratic' or 'not sagainst each of these.

(a) Country A: People who do not accept the country's official religion do not havright to vote
(b) Country B: The same party has been winning elections for the last twenty yea(c) Country C: Ruling party has lost in the last three elections
(d) Country D:
There is no independent election commission

Question 2: Here is some information about four countries. Based on this information, how would you classify each of these countries. Write 'democratic', 'undemocratic' or 'not sure' against each of these.

(a) Country P: The parliament cannot pass a law about the army without the consent of the Chief of Army
(b) Country Q: The parliament cannot pass a law reducing the powers of the judiciary
(c) Country R: The country's leaders cannot sign any treaty with another country without taking permission from its neighbouring country.
(d) Country S: All the major economic decisions about the country are taken by officials of the central bank which the ministers cannot change.

Question 3: Which of these is not a good argument in favour of democracy? Why?

(a) People feel free and equal in a democracy
(b) Democracies resolve conflict in a better way than others
(c) Democratic government is more accountable to the people
(d) Democracies are more prosperous than others

Question 4: Each of these statements contains a democratic and an undemocratic element. Write out the two separately for each statement.

(a) A minister said that some laws have to be passed by the parliament in order to conform to the regulations decided by the World Trade Organisation.
(b) The Election Commission ordered re-polling in a constituency where large scale rigging was reported.
(c) Women's representation in the parliament has never reached 10 per cent. This led women's organisations to demand one-third seats for women.

Question 5: Which of these is not a valid reason for arguing that there is a lesser possibility of famine in a democratic country?

(a) Opposition parties can draw attention to hunger and starvation.
(b) Free press can report suffering from famine in different parts of the country.
(c) Government fears its defeat in the next elections.
(d) People are free to believe in and practise any religion.

Question 6: There are 40 villages in a district where the government has made no provision fordrinking water. These villagers met and considered many methods of forcing thegovernment to respond to their need. Which of these is not a democratic method?

(a) Filing a case in the courts claiming that water is part of right to life.
(b) Boycotting the next elections to give a message to all parties.
(c) Organising public meetings against government's policies.
(d) Paying money to government officials to get water.

Question 7: Write a response to the following arguments against democracy.

(a) Army is the most disciplined and corruption-free organisation in the country. Therefore army should rule the country.
(b) Rule of the majority means the rule of ignorant people. What we need is the rule of the wise, even if they are in small numbers.
(c) If we want religious leaders to guide us in spiritual matters, why not invite them to guide us in politics as well. The country should be ruled by religious leaders.

Question 8: Are the following statements in keeping with democracy as a value? Why?

(a) Father to daughter: I don't want to hear your opinion about your marriage. Inour family children marry where the parents tell them to.
(b) Teacher to student: Don't disturb my concentration by asking me questions inthe classroom.
(c) Employee to the officer: Our working hours must be reduced according to thelaw.

Question 9:  Consider the following facts about a country and decide if you would call it ademocracy. Give reasons to support your decision.

(a) All the citizens of the country have right to vote Elections are held regularly.
(b) The country took loan from international agencies. One of the conditions forgiven loan was that the government would reduce its expenses on education andhealth.
(c) People speak more than seven languages but education is available only in onelanguage, the language spoken by 52 percent people of the country.
(d) Several organisations have given a call for peaceful demonstrations and nation wide strikes in the country to oppose these policies. Government has arrested these leaders.
(e) The government owns the radio and television in the country. All the newspapers have to get permission from the government to publish any news about government's policies and protests.

Question 10: In 2004 a report published in USA pointed to the increasing inequalities in thatcountry. Inequalities in income reflected in the participation of people in democracy...


(Political Science) Chapter 3 Constitutional Design


Question 1: Here are some false statements. Identify the mistake in each case and rewrite these correctly based on what you have read in this chapter.

(a) Leaders of the freedom movement had an open mind about whether the country should be democratic or not after independence.
(b) Members of the Constituent Assembly of India held the same views on all provisions of the Constitution.
(c) A country that has a constitution must be a democracy.
(d) Constitution cannot be amended because it is the supreme law of a country.

Question 2: Which of these was the most salient underlying conflict in the making of a democratic constitution in South Africa?

(a) Between South Africa and its neighbours
(b) Between men and women
(c) Between the white majority and the black minority
(d) Between the coloured minority and the black majority

Question 3: Which of these is a provision that a democratic constitution does not have?

(a) Powers of the head of the state
(b) Name of the head of the state
(c) Powers of the legislature
(d) Name of the country

Question 4: Match the following leaders with their roles in the making of the Constitution:

Question 5: Read again the extracts from Nehru's speech "Tryst with Destiny" and answer the following:

(a) Why did Nehru use the expression "not wholly or in full measure" in the first sentence?
(b) What pledge did he want the makers of the Indian Constitution to take?
(c) "The ambition of the greatest man of our generation has been to wipe every tear from every eye". Who was he referring to?

Question 6: Here are some of the guiding values of the Constitution and their meaning. Rewrite them by matching them correctly.

Question 7: A friend from Nepal has written you a letter describing the political situation there. Many political parties are opposing the rule of the king. Some of them say that the existing constitution given by the monarch can be amended to allow more powers to elected representatives. Others are demanding a new Constituent Assembly to write a republican constitution. Reply to your friend giving your opinions on the subject.

Question 8: Here are different opinions about what made India a democracy. How much importance would you give to each of these factors?

(a) Democracy in India is a gift of the British rulers. We received training to work with representative legislative institutions under the British rule.
(b) Freedom struggle challenged the colonial exploitation and denial of different freedoms to Indians. Free India could not be anything but democratic.
(c) We are lucky to have leaders who had democratic convictions. The denial of democracy in several other newly independent countries shows the important role of these leaders.

Question 9: Read the following extract from a conduct book for 'married women', published in 1912. 'God has made the female species delicate and fragile both physically and emotionally, pitiably incapable of self-defence. They are destined thus by God to remain in male protection − of father, husband and son − all their lives. Women should, therefore, not despair, but feel obliged that they can dedicate themselves to the service of men'. Do you think the values expressed in this para reflected the values underlying our constitution? Or does this go against the constitutional values?

Question 10: Read the following statements about a constitution. Give reasons why each of these is true or not true.

(a) The authority of the rules of the constitution is the same as that of any other law.
(b) Constitution lays down how different organs of the government will be formed.
(c) Rights of citizens and limits on the power of the government are laid down in the constitution.
(d) A constitution is about institutions, not about values.


(Political Science) Chapter 4 Electoral Politics


Question 1: Which of the following statements about the reasons for conducting elections are false?

(a) Elections enable people to judge the performance of the government.
(b) People select the representative of their choice in an election.
(c) Elections enable people to evaluate the performance of the judiciary.
(d) People can indicate which policies they prefer.

Question 2: Which of these is not a good reason to say that Indian elections are democratic?

(a) India has the largest number of voters in the world.
(b) India's Election Commission is very powerful.
(c) In India, everyone above the age of 18 has a right to vote.
(d) In India, the losing parties accept the electoral verdict.

Question 3: Match the following:

Question 4: List all the different election related activities mentioned in the chapter and arrange them in a time sequence, beginning with the first activity and ending with the last. Some of these activities are given below: releasing election manifestos; counting of votes; making of voters' list; election campaign; declaration of election results; casting of votes; ordering of re-poll; announcing election schedule; filing nomination.

Question 5: Surekha is an officer in-charge of ensuring free and fair elections in an assembly constituency in a state. Describe what should she focus on for each of the following stages of election:

(a) Election campaign
(b) Polling day
(c) Counting day

Question 6: The table below gives the proportion of different communities among the candidateswho won elections to the US Congress. Compare these to the proportion of thesecommunities in the population of the US. Based on this, would you suggest a systemof reservations in the US Congress? If yes, why and for which communities? If no,why not.

Question 7: Can we draw the following conclusions from the information given in this chapter? Give two facts to support your position for each of these.

(a) Election Commission of India does not have enough powers to conduct free and fair elections in the country.
(b) There is a high level of popular participation in the elections in our country.
(c) It is very easy for the party in power to win an election.
(d) Many reforms are needed to make our elections completely free and fair.

Question 8: Chinappa was convicted for torturing his wife for dowry. Satbir was held guilty of practicing untouchability. The court did not allow either of them to contest elections. Does this decision go against the principles of democratic elections?

Question 9: Here are some reports of electoral malpractices from different parts of the world. Is there anything that these countries can learn from India to improve their elections? What would you suggest in each case?

(a) During an election in Nigeria, the officer in charge of counting votes deliberately increased the votes of one candidate and declared him elected. The court later found out that more than five lakh votes cast for one candidate were counted in favour of
another.
(b) Just before elections in Fiji, a pamphlet was distributed warning voters that a vote for former Prime Minisiter, Mahendra Chaudhry will lead to bloodshed. This was a threat to voters of India origin.
(c) In the US, each state has its own method of voting, its own procedure of counting and its own authority for conducting elections. Authorities in the state of Florida took many controversial decisions that favoured Mr. Bush in the presidential elections in 2000. But no one could change those decisions.

Question 10: Here are some reports of malpractices in Indian elections. Identify what the problem in each case is. What should be done to correct the situation?

(a) Following the announcement of elections, the minister promised to provide financial aid to reopen the closed sugar mill.
(b) Opposition parties alleged that their statements and campaign was not given due attention in Doordarshan and All India Radio.
(c) An inquiry by the Election Commission showed that electoral rolls of a state contain name of 20 lakh fake voters.
(d) The hoodlums of a political party were moving with guns, physically preventing supporters of other political parties to meet the voters and attacking meetings of other parties.

Question 11: Ramesh was not in class when this chapter was being taught. He came the next day and repeated what he had heard from his father. Can you tell Ramesh what is wrong with these statements?

(a) Women always vote the way men tell them to. So what is the point of giving them the right to vote?
(b) Party politics creates tension in society. Elections should be decided by consensus not by competition.
(c) Only graduates should be allowed to stand as candidates for elections.


(Political Science) Chapter 5 Working of Institutions


Question 1: If you are elected as the President of India which of the following decisions can you take on your own?

(a) Select the person you like as Prime Minister.
(b) Dismiss a Prime Minister who has a majority in Lok Sabha.
(c) Ask for reconsideration of a bill passed by both the Houses.
(d) Nominate the leaders of your choice to the Council of Ministers.

Question 2: Who among the following is a part of the political executive?

(a) District Collector
(b) Secretary of the Ministry of Home Affairs
(c) Home Minister
(d) Director General of Police

Question 3: Which of the following statements about the judiciary is false?

(a) Every law passed by the Parliament needs approval of the Supreme Court.
(b) Judiciary can strike down a law if it goes against the spirit of the Constitution.
(c) Judiciary is independent of the Executive.
(d) Any citizen can approach the courts if her rights are violated

Question 4: Which of the following institutions can make changes to an existing law of the country?

(a) The Supreme Court
(b) The President
(c) The Prime Minister
(d) The Parliament

Question 5: Match the ministry with the news that the ministry may have released:

Question 6: Of all the institutions that we have studied in this chapter, name the one thatexercises the powers on each of the following matters.

(a) Decision on allocation of money for developing infrastructure like roads, irrigationetc. and different welfare activities for the citizens.
(b) Considers the recommendation of a Committee on a law to regulate the stockexchange.
(c) Decides on a legal dispute between two state governments.
(d) Implements the decision to provide relief for the victims of an earthquake.

Question 7: Why is the Prime Minister in India not directly elected by the people? Choose the most appropriate answer and give reasons for your choice.

(a) In a Parliamentary democracy only the leader of the majority party in the Lok Sabha can become the Prime Minister.
(b) Lok Sabha can remove the Prime Minister and the Council of Ministers even before the expiry of their term.
(c) Since the Prime Minister is appointed by the President there is no need for it.
(d) Direct election of the Prime Minister will involve lot of expenditure on election.

Question 8: Three friends went to watch a film that showed the hero becoming Chief Minister for a day and making big changes in the state. Imran said this is what the country needs. Rizwan said this kind of a personal rule without institutions is dangerous. Shankar said all this is a fantasy. No minister can do anything in one day. What would be your reaction to such a film?

Question 9: A teacher was making preparations for a mock parliament. She called two students to act as leaders of two political parties. She gave them an option: Each one could choose to have a majority either in the mock Lok Sabha or in the mock Rajya Sabha. If this choice was given to you. Which one would you choose and why?

Question 10: After reading the example of the reservation order, three students had different reactions about the role of the judiciary. Which view, according to you, is a correct reading of the role of judiciary?

(a) Srinivas argues that since the Supreme Court agreed with the government, it is not independent.
(b) Anjaiah says that judiciary is independent because it could have given a verdict against the government order. The Supreme Court did direct the government to modify it.
(c) Vijaya thinks that the judiciary is neither independent nor conformist, but acts as a mediator between opposing parties. The court struck a good balance between those who supported and those who opposed the order.


(Political Science) Chapter 6 Democratic Rights


Question 1: Which of the following is not an instance of an exercise of a fundamental right?

(a) Workers from Bihar go to the Punjab to work on the farms
(b) Christian missions set up a chain of missionary schools
(c) Men and women government employees get the same salary
(d) Parents' property is inherited by their children

Question 2: Which of the following freedoms is not available to an Indian citizen?

(a) Freedom to criticise the government
(b) Freedom to participate in armed revolution
(c) Freedom to start a movement to change the government
(d) Freedom to oppose the central values of the Constitution

Question 3: Which of the following rights is available under the Indian Constitution?

(a) Right to work
(b) Right to adequate livelihood
(c) Right to protect one's culture
(d) Right to privacy

Question 4: Name the Fundamental Right under which each of the following rights falls:

(a) Freedom to propagate one's religion
(b) Right to life
(c) Abolition of untouchability
(d) Ban on bonded labour

Question 5: Which of these statements about the relationship between democracy and rights is more valid? Give reasons for your preference.

(a) Every country that is a democracy gives rights to its citizens.
(b) Every country that gives rights to its citizens is a democracy.
(c) Giving rights is good, but it is not necessary for a democracy.

Question 6: Are these restrictions on the right to freedom justified? Give reasons for your answer.

(a) Indian citizens need permission to visit some border areas of the country for reasons of security.
(b) Outsiders are not allowed to buy property in some areas to protect the interest of the local population.
(c) The government bans the publication of a book that can go against the ruling party in the next elections.

Question 7: Manoj went to a college to apply for admission into an MBA course. The clerk refused to take his application and said "You, the son of a sweeper, wish to be a manager! Has anyone done this job in your community? Go to the municipality office and apply for a sweeper's position". Which of Manoj's fundamental rights are being violated in this instance? Spell these out in a letter from Manoj to the district collector.

Question 8: When Madhurima went to the property registration office, the Registrar told her. "You can't write your name as Madhurima Banerjee d/o Α. K. Banerjee. You are married, so you must give your husband's name. Your husband's surname is Rao. So your name should be changed to Madhurima Rao." She did not agree. She said "If my husband's name has not changed after marriage, why should mine?" In your opinion who is right in this dispute? And why?

Question 9:  Thousands of tribals and other forest dwellers gathered at Piparia in Hoshangabad district in Madhya Pradesh to protest against their proposed displacement from the Satpura National Park, Bori Wildlife Sanctuary and Panchmarhi Wildlife Sanctuary. They argue that such a displacement is an attack on their livelihood and beliefs. Government claims that their displacement is essential for the development of the area and for protection of wildlife. Write a petition on behalf of the forest dwellers to the NHRC, a response from the government and a report of the NHRC on this matter.

Question 10: Draw a web interconnecting different rights discussed in this chapter. For example right to freedom of movement is connected to the freedom of occupation. One reason for this is that freedom of movement enables a person to go to place of work within one's village or city or to another village, city or state. Similarly this right can be used for pilgrimage, connected with freedom to follow one's religion. Draw a circle for each right and mark arrows that show connection between or among different rights. For each arrow, give an example that shows the linkage.


(Geography) Chapter 1 India - Size and Location


Problem 1. Choose the right

(i) Tropic of Cancer does not pass through

(a) Rajasthan
(b) Orissa
(c) Chhattisgarh
(d) Tripura

Solution : (b)

(ii) The easternmost longitude of India is

(a) 97° 25′ E
(b) 68° 7′ E
(c) 77° 6′ E
(d) 82° 32′ E

Solution : (a)

(iii) Uttarakhand, Uttar Pradesh, Bihar, West Bengal and Sikkim have common frontier with

(a) China
(b) Nepal
(c) Bhutan
(d) Myanmar

Solution : (c)

(iv) If you intended to visit Kavaratti during your summer vacations, which one of the following Union Territories of India will you be going to?

(a) Puducherry
(b) Lalcshadweep
(c) Bangladesh
(d) Nepal

Solution : (b)

(v) My friend hails from a country which does not share land boundary with India. Identify the country.

(a) Bhutan
(b) Bangladesh
(c) Tajikistan
(d) Nepal

Solution : (b)

Problem 2. the followings briefly.

(i) Name the group of islands lying in the Arabian Sea.

Solution : Lakshadweep Islands lie in the Arabian Sea.

(ii) Name the countries which are larger than India.

Solution : Russia, Canada, USA. China, Australia and Brazil are the countries that are larger than India.

(iii) Which island group of India lies to its South-East?

Solution : Andaman and Nicobar Islands lie to the South-East of India.

(iv) Which island countries are our Southern neighbours?

Solution :
Sri Lanka and Maldives are the two island countries that are the Southern neighbours of India.

Problem 3. The Sun rises two hours earlier in Arunachal Pradesh as compared to Gujarat in the West but the watches show the same time. How does this happen?

Solution :
From Gujarat to Arunachal Pradesh there is a time lag of two hours, but the watches show the same time because the time along the Standard Meridian of India (82 0 30′ E) passing through Mirzapur in Uttar Pradesh is taken as the standard time for the whole country. Because the same standard time for the whole country has been adopted, the watches show the same time in Arunachal Pradesh and Gujarat and in all parts of the country.

Problem 4. The central location of India at the head of the Indian Ocean is considered of great significance. Why?

Solution :
The Indian landmass has a central location between East and West Asia. India is a southward extension of the Asian continent. The trans Indian Ocean routes which connect the countries of Europe in the West to the countries of East Asia provide a strategic central location to India. The Deccan peninsula protrudes into the Indian Ocean, thus helping India to establish close contact with West Asia, Africa and Europe from the Western coast and South-East and East Asia from the Eastern coast. No other country has a long coastline on the Indian Ocean as India has. Thus, India's eminent position in the Indian Ocean Justifies the naming of an ocean after it. Map Skills

Problem 1. Identify the following with the help of map reading.

(i) The island groups of India lying in the Arabian Sea and the Bay of Bengal.

Solution : Lakshadweep Islands and Andaman and Nicobar Islands

(ii) The countries constituting the Indian Sub-continent.

Solution : Pakistan, Afghanistan, Nepal, Bhutan, Myanmar and Bangladesh.

(iii) The states through which the Tropic of Cancer passes.

Solution :
Gujarat, Rajasthan, Madhya Pradesh, Chhattisgarh, Jharkhand, Paschim Banga, Tripura and Mizoram.

(iv) The northernmost latitude in degrees.

Solution :
37° 6'N

(v) The southernmost latitude of the Indian mainland in degrees.

Solution :
8° 4'N

(vi) The eastern and the Westernmost longitude in degrees.

Solution :
68° 7'E and 97° 25'E

(vii) The place situated on the three seas.

Solution :
Kanyakumari

(viii) The straight separating Sri Lanka from India.

Solution :
Palk Strait

(ix) The Union Territories of India.

Solution :
Andaman and Nicobar Islands, Chandigarh, Dadra and Nagar Haveli, Daman and Diu, Lakshadweep, Puducherry and Delhi. Project/Activity

(i) Find out the longitudinal and latitudinal extent of your state.

Solution :
is given below My state is Uttar Pradesh longitudinally it extends from 77°4'E to 84°39'E. Latitudinally it extends from about 30 0 20'N to about 23°53'N.

(ii) Collect information about the 'Silk Route Also find out the new developments which are improving communication routes in the regions of the high altitude.

Solution :
(a) Do it yourself. It is given in your History book. Some information is given below

The Silk Road or Silk Route refers to a historical network of interlinking trade routes across the Afro-Eurasian landmass that connected East, South. and Western Asia with the Mediterranean and European world, as well as parts of North and East Africa.

The land routes were supplemented by sea routes, which extended from the Red Sea to coastal India, China and South-East Asia.

Extending 4,000 miles (6,500 km), the Silk Road gets its name from the lucrative Chinese silk trade along it. which began during the Han Dynasty (206 BCE – 220 CE). The central Asian sections of the trade routes were expanded around 114 BCE by the Han dynasty largely through the missions and explorations of Zhang Qian, but earlier trade routes across the continents already existed.

In the late Middle Ages. transcontinental trade over the land routes of the Silk Road declined as sea trade increased. In recent years, both the maritime and overland Silk Routes are again being used, often closely following the ancient routes.

(b) Communication routes in regions of high altitude are now-a-days through helicopter (for closer distances) and through high altitude railways like the Ouinghai railway in China linking Lhasa (capital of Tibet) with the rest of the world. More information can be found on this railway from the internet.


(Geography) Chapter 2 Physical Features of India


Problem 1. Choose the right Solution from the four alternatives given below.

(i) A landmass bounded by sea on three sides in referred to as

(a) coast
(b) island
(c) peninsula
(d) None of these

Solution : (c)

(ii) Mountain ranges in the eastern part of India forming its boundary with Myanmar are collectively called as

(a) Himachal
(b) Uttarakhand
(c) Purvanchal
(d) None of these

Solution : (c)

(iii) The Western coastal strip, South of Goa is referred to as

(a) Coromandel
(b) Konkan
(c) Kannad
(d) Northern Circar

Solution : (c)

(iv) The highest peak in the Eastern Ghats is

(a) AnaMudi
(b) Kanchenjunga
(c) Mahendragiri
(d) Khasi

Solution : (c)

Problem 2. the following Questions briefly

(i) What are tectonic plates ?

Solution : The convectional currents below the Earth's crust split the crust into a number of pieces. These pieces are called tectonic plates. e.g., Eurasia plate, Indo Australian plate etc.

(ii) Which continents of today were part of the Gondwanaland ?

Solution : The Gondwana land included India, Australia, South Africa and South America as one single land mass.

(iii) What is the Bhabar?

Solution :
Bhabar is a narrow belt of land about 8 to 16 km wide and covered with pebbles deposited by the rivers and lying parallel to the slopes of the Shiwaliks

(iv) Name the three major divisions of the Himalayas from north to south.

Solution : The 3 major divisions of the Himalayas from North to South are as folloWS

(a) The Great or Inner Himalayas or the Himadri It is a continuous range consisting of the highest peaks.
(b) Himachal or Lesser Himalayas The range lying to the South of the Himadri is known as Himachal or Lesser Himalayas.
(c) Shiwaliks The outermost range of the Himalayas is known as the Shiwaliks. These are the foothill ranges and represent the southernmost division of the Himalayas.

(v) Which plateaus lies between the Aravalli and the Vindhyan ranges?

Solution : Malwa plateau lies between the Aravalli and the Vindhyan ranges.

(vi) Name of the Island group of India having coral Vindhyan

Solution : Lakshadweep Islands are composed of small coral islands.

Problem 3. Distinguish between

(i) Converging and diverging tectonic plates.

Solution : Difference between Converging plates and Diverging plates

(ii) Bhangar and Khadar.

Solution : Differences between Bhangar and Khadar

(iii) Western Ghats and Eastern Ghats.

Solution : Difference between the Eastern Ghats and the Western Ghats.

Problem 4. Describe how the Himalayas were formed.

Solution :

(a) The oldest landmass of India (peninsular part) was a part of Gondwana land.
(b) Gondwana land included India, Australia, South Africa and South America as one single landmass.
(c) Convectional currents split the crust into a number of pieces, thereby leading to the drifting of the Indo – Australian plate after being separated from the Gondwana land towards the North.
(d) The Northward drift resulted in the collision of the plate with the much larger Eurasian plate.
(e) Due to this collision the sedimentary rocks which were accumulated in the geosyncline known as the Tethys, were folded to form the mountain systems of Western Asia and Himalaya.
(f) The Himalayas represent a youthful topography with high peaks, deep valleys and fast following rivers.

Problem 5. Which are the major physiographic divisions of India? Contrast the relief of the Himalayan region with that of the Peninsular plateau.

Solution :
The major physiographic divisions of India are

The Himalayan mountains
The Northern plains
The Peninsular plateau
The Indian desert
The Coastal plains
The islands

Contrast between the Himalayan region and the Peninsular plateau

Problem 6. Give an account of the Northern plains of India.

Solution : (a) The Himalayan uplift out of the Tethys sea and subsidence of the Northern flank of the peninsular plateau resulted in the formation of a large basin. In course of time, this depression gradually got filled with deposition of sediments from rivers flowing from the mountains In the north and led to formation of the fertile Northern plains.
(b) It spreads over an area of 7 lakh sq km. The plain IS about 2400 km long and 240-230 km broad
(c) It is a densely populated and an Intensively cultivated area.
(d) With Its adequate water supply and favourable climate, It is agriculturally a very productive part of India
(e) The Northern plains are broadly divided into 3 sections.
(f) The Western part of the Northern plans IS called the Punjab plains, formed by the Indus and its tributaries.
(g) The Ganga plain extends between the Ghaggar and the Teesta rivers, spread over the states of Haryana, Delhi, UP Bihar and West Bengal.
(h) The Brahmaputra Plain to the East of the Ganga plains lies the Brahmaputra plain covering the areas of Assam and Arunachal
Pradesh

Problem 7. Write short notes on the following

(i) The Indian Desert
(ii) The Central Highlands
(iii) The Island Groups of India

Solution : (i) The Indian Desert
The Indian desert lies towards the Western margin of the Aravalli hills. It is an undulating sandy plain covered with sand dunes.
This region receives very low rainfall, below 150 mm per year. It has arid climate with low vegetation cover. Streams appear during the rainy season. Soon they disappear into the sand as they do not have enough water to reach the sea. Luni is the only large river in this region. Barchan (Crescent shaped dunes) cover the larger areas but longitudinal dunes become more prominent near the Indo – Pakistan border.
The Indian desert is popularly known as the Thar desert.

(ii) The Central Highlands The Peninsular plateau consists of two broad divisions, namely the central highlands and the Deccan plateau.
The part of the Peninsular plateau lying to the north of the Narmada river covering a major area of the Malwa plateau is known as the Central Highlands.
They are bound by the Vindhya Range from the south and by the Aravalli hills from the north-west.
The further westward extension gradually merges with the sandy and rocky desert of Rajasthan.
The flow of the rivers draining this region, namely the Chambal, the Sind, the Betwe and Ken is from south-west to north-east.
The central highlands are wider in the west but narrower in the east.
The eastward extensions of their plateau are locally known as 'Bundelkhand' and Baghelkhand.
The Chhotanagpur plateau marks the further eastward extension drained by the Damodar river.

(iii) The Island Groups of India India has two groups of islands namely the Lakshadweep group and the Andaman and Nicobar group.

(a) The Lakshadweep Islands Group lie close to the Malabar coast of Kerala.
These islands are composed of small coral islands.
Earlier, they were known as Laccadive, Minicoy and Amindive. In 1973, these were renamed as the Lakshadweep.
They cover a small area of 32 sq km.
Kavaratti Island is the administrative headquarters of Lakshadweep.
This island group has a great diversity of flora and fauna.
The Pitti Island, which is uninhabited, has a bird sanctuary.

(b) Andaman and Nicobar Islands
The elongated chain of islands located in the Bay of Bengal extending from north to south are the Andaman and Nicobar Islands.
They are bigger in size and more numerous and scattered than the Lakshadweep Islands.
The entire group of islands is divided into two broad categories, the Andaman in the north and Nicobar in the south.
It is believed that these islands are an elevated portion of the submarine mountains.
These islands are of great – strategic importance for the country.
They have great diversity of flora and fauna.
These islands lie close to the equator experience equatorial climate and have thick forest cover.

Project / Activity

Locate the peaks, passes, ranges, plateaus hills and duns hidden in the puzzle.
Try to find where these features are located. You may start your search horizontally vertically or diagonally.

Solution :

  • Horizontally the features ale

  • Nathula

  • Cardamom

  • Garo

  • Kanchenjunga

  • Anamudi

  • Everest

  • Patli

  • Vertically the features are

  • Chottanagpur

  • Aravali

  • Konkan

  • Jaintia

  • Malwa

  • Nilgirl

  • Shipki La

  • Vindhya

  • Bomdila

  • Sahyadri

  • Satpura (this one is from bottom to top)


(Geography)  Chapter 3 Drainage


Problem 1. Choose the right Solution from the four alternatives given below

(i) Which one of the following describes the drainage patterns resembling the branches of a tree?

(a) Radial
(b) Dendritic
(c) Centrifugal
(d) Trellis

Solution (b)

(ii) In which of the following states is the Wular Lake located ?

(a) Rajasthan
(b) Punjab
(c) Uttar Pradesh
(d) Jammu and Kashmir

Solution (d)

(iii) The river Narmada has its source as

(a) Satpura
(b) Amarkantak
(c) Brahmagiri
(d) Slopes of the Western Ghats

Solution (b)

(iv) Which one of the following lakes is a salt water lake?

(a) Sambhar
(b) Wular
(c) Dal
(d) Gobind Sagar

Solution (a)

(v) Which one of the following is the longest river of peninsular India?

(a) Narmada
(b) Krishna
(c) Godavari
(d) Mahanadi

Solution (c)

(vii) Which one amongst the following rivers flow through a Rift Valley?

(a) Mahanadi
(b) Krishna
(c) Tungabhadra
(d) Tapi

Solution (d)

Problem 2. Solution the following Problem s briefly.

(i) What is meant by a water divide? Give an example

Solution Any elevated area such as a mountain or an upland that separate two drainage basins is called a water divide. An example are the Western Ghats.

(ii) Which is the largest river basin in India?

Solution The Ganga Basin is the largest river basin in India.

(iii) Where do the rivers Indus and Ganga have their origin?

Solution The river Indus rises in Tibet. near lake Mansarovar. and the Ganga originates at the Gangotri Glacier. Both of them have their origin in the Himalayas.

(iv) Name the two headstreams of the Ganga. Where do they meet to form the Ganga?

Solution The two head streams of the Ganga are the Bhagirathi and Alaknanda. They meet at Devprayag in Uttarakhand to form the Ganga.

(v) Why does the Brahmaputra in its Tibetan part have less silt, despite a longer course?

Solution In TIbet. the river Indus known as Tsang Po carries a smaller volume of water and less silt as it is a cold and dry area.
In India. the river carries a large volume of water and considerable amount of silt because it passes through a region of high rainfall.

(vi) Which two peninsular rivers flow through a trough ?

Solution Narmada and Tapi are two peninsular rivers, which flow through a trough.

(vii) State some economic benefits of rivers and lakes.

Solution
Lakes

Lakes are of a great value to human beings.
Lakes help to regulate the flow of rivers.
Lakes help to prevent flooding during rainy season.
During the dry season, lakes help to maintain an even flow of water.
Lakes can also be used for developing hydel power.
They moderate the climate of the surroundings and maintain the aquatic ecosystem.
They enhance natural beauty and help to develop tourism and provide recreation. e.g., Dal Lake and Naini Lake at Nainital.
Lakes provide opportunities for fishery development.

Rivers

They help to develop hydel power.
They provide water for irrigation, for drinking and other requirements.
They help to develop fisheries.


(Geography) Chapter 4 Climate


Problem 1. Choose the correct Solution from the four alternatives given below

(i) Which one of the following places receives the highest rainfall in the world?

(a) Silchar
(b) Mawsynram
(c) Cherrapunji
(d) Guwahati

Solution : (b)

(ii) The wind blowing in the northern plains in summers is known as

(a) Kaal Baisakhi
(b) Loo
(c) Trade Winds
(d) None of these

Solution : (b)

(iii) Which one of the following causes rainfall during winters in north-western part of India?

(a) Cyclonic depression
(b) Western disturbances
(c) Retreating monsoon
(d) Southwest monsoon

Solution : (a)

(iv) Monsoon arrives in India approximately

(a) early May
(b) early July
(c) early June
(d) early August

Solution : (c)

(v) Which one of the following characterises the cold weather season in India?

(a) Warm days and warm nights
(b) Warm days and cold nights
(c) Cool days and cold nights
(d) Cool days and warm nights

Solution : (b)

Problem 2. Solution the following Problems briefly.

(i) What are the controls affecting the climate of India?

Solution : There are six major controls of the climate of any place. They are latitude, altitude, pressure and wind system, distance from the sea (continentality), ocean currents and relief features.

(ii) Why does India have a monsoon type of climate?

Solution : The monsoon winds play an important role in the climate of India. Therefore, it is called the monsoon type of climate.

(iii) Which part of India does experience the highest diurnal range of temperature and why?

Solution : The north-western part of India experiences the highest diurnal range of temperature. In the Thar desert, the day temperature may rise to 50 0 e and drop down to near 15°e the same night. On the other hand, there is hardly any difference in day and night temperatures in the Andaman and Nicobar Islands or in Kerala .

(iv) Which winds account for rainfall along the Malabar coast?

Solution : The south west monsoon winds are responsible for the rainfall along the Malabar coast.

(v) What are jet streams and how do they affect the climate of India?

Solution : Jet streams are a narrow belt of high altitude (above 12000 m) westerly winds in the troposphere.

(a) Their speed varies from about 110 km/h in summer to about 184 km/h in winter.
(b) A number of separate jet streams have been identified.
(c) The most constant are the mid latitude and subtropical jet streams.
(d) Jet streams over the Indian peninsula during the summer affect the monsoon.
(e) The subtropical westerly jet stream blow south of the Himalayas and is responsible for the western cyclonic disturbances experienced in the north and north western parts of the country.
(f) An easterly jet stream blows over peninsular India. It affects the coastal regions of the country and is responsible for tropical cyclones during the monsoon as well as during the October to November period.

(vi) Define monsoons. What do you understand by 'break' in monsoon?

Solution : The monsoons are moisture laden winds from the southwest which bring heavy rainfall to southern Asia, in summer.

'Break' in monsoon means that the monsoon has alternate wet and dry spells. This means that the monsoon rains take place for a few days at a time. These wet spells are interspersed with dry spells related to the movement of the monsoon trough.

(vii) Why is monsoon considered a unifying bond?

Solution : The seasonal alteration of the wind systems and the associated weather conditions provide a rhythmic cycle of seasons.

Monsoon rains are unevenly distributed and typically uncertain. The Indian landscape, plant and animal life, agriculture, the people and their festivities, all revolve around the monsoon.

All the Indian people eagerly await the arrival of the monsoon. It binds the whole country by providing water which sets all agricultural activities in motion. That is why the monsoon is considered a unifying bond.

Problem 3. Why does rainfall decrease from the east to the west in northern India?

Solution : Rainfall decreases from the east to the west in Northern India because there is a decrease in the moisture of the winds. As the moisture bearing winds of the Bay of Bengal branch of the south west monsoon move further and further inland, the moisture gradually decreases and results in low rainfall when moving westwards. Consequently, states like Gujarat and Rajasthan in western India get very little rainfall.

Problem 3. Give reasons as to why

(i) Seasonal reversal of wind direction takes place over the Indian subcontinent.
(ii) The bulk of rainfall in India is concentrated over a few months.
(iii) The Tamil Nadu coast receives winter rainfall.
(iv) The delta region of the eastern coast is frequently struck by cyclones.
(v) Parts of Rajasthan, Gujarat and the leeward side of the Western Ghats are drought prone.

Solution :
(i) During winter, there is a high pressure area north of the Himalayas. Cold winds blow from this region to the low pressure areas over the oceans to the south.

(a) In summer, a low pressure area develops over interior Asia as well as over north-western India.
(b) This causes a complete reversal of the direction of winds during summer.


(ii) In summer, a low pressure area develops over interior Asia as well as over north western India

(a) This causes a complete reversal of the direction of winds during summer. Air moves from the high pressure area over the southern Indian ocean, crosses the equator and turns right towards the low pressure areas over the Indian subcontinent
(b) These are known as the south-west monsoon winds
(c) These winds blow over warm oceans, gather moisture and bring widespread rainfall over the mainland of India
(d) The duration of the monsoon is between 100-120 days from early June to mid September. Thus, we can say that rainfall in India is concentrated over a few months.

(iii) During the winter season, -the north-west trade winds prevail over the country. They blow from land to sea and hence for most part of the country it is a dry season. Some amount of rainfall occurs on the Tamil Nadu coast from these winds as here they blow from sea to land.

(iv) The delta region of the eastern coast is frequently struck by cyclones because the cyclonic depressions which originate over the Andaman Sea generally cross the Eastern coasts of India and cause heavy and widespread rain.

(a) These cyclones are often very destructive. The thickly populated deltas of the Godavari, the Krishna and the Kaveri are frequently struck by cyclones which cause great damage to life and property.
(b) Sometimes these cyclones arrive at the coasts of Odisha, West Bangal and Bangladesh.

(v) Parts of Rajasthan, Gujarat and the leeward side of the Western Ghats are drought prone because they receive scanty rainfall. Even during the monsoon months the monsoon winds when rising over the Western Ghats give rain to that area. By the time they reach Rajasthan and Gujarat there is very less moisture left in these winds and so these areas are drought prone.

Problem 4. Describe the regional variations in the climatic conditions of India with the help of suitable examples.

Solution : There are regional variations in the climatic conditions of India which can be understood with the help of the following examples

(a) The months of December and January are the coldest in Northern India where the temperature ranges between 10°-15°.
(b) In summer, the mercury occasionally touches 50°C in some parts of the Rajasthan desert, whereas it might be around 2O°C in pahalgam In Jammu and Kashmir.
(c) On a winter night temperature at Drass in Jammu and Kashmir may be as low as minus 40″C. Tiruvananthapuram on the other hand may have a temperature of 22° C.
(d) Annual precipitation varies from over 400 em in Meghalaya to less than 10 cm in Ladakh and western Rajasthan. In the Himalayas precipitation is in the form of snowfall.
(e) Coastal region does not experience much variation in temperature pattern due to the moderating influence of the sea.

Problems 5. Discuss the mechanism of monsoon.

Solution : The climate of India is described as the monsoon type

(a) The factors affecting the climate of an area are latitude, altitude, pressure and wind system distance from the sea
(b) Pressure and surface winds, and relief features
(c) India lies in the region of north-easterly winds. These winds originate from the subtropical high pressure belt in the northern hemisphere, get deflected to the right due to the coriolls force and move on towards the equatorial low pressure area.
(d) In summer, a low pressure area develops over interior Asia andnorth-western India. This causes complete reversal of the direction of the winds during summer. Air moves from the high pressure area over the southern Indian Ocean, crosses the equator and turns right towards the low pressure areas over the Indian subcontinent. These are known as the south-west monsoon winds. These winds blow over the warm oceans, gather moisture and bring widespread rainfall over the mainland of India.
(e) The upper air circulation in this region is dominated by a westerly flow.
(f) The duration of the monsoon is between 100-120 days from early June to mid September
(g) The Southern Oscillation (SO) and jet streams also affects the monsoon.Withdrawal or Retreat of Monsoon The withdrawal or retreat of the monsoon begins in the states of India by early September. By mid October, it withdraws completely from the northern half of the peninsula. By December, the monsoon has withdrawn from the rest of the country.

Problem 6. Give an account of weather conditions and characteristics of the cold season.

Solution : The cold weather season begins from the November in northern India and stays till February. December and January are the coldest onths in the northern part of India.

(a) The weather is normally marked by clear sky, low temperatures, low humidity and feeble variable winds
(b) Days are warm and nights are cold. Frost is common in the north and higher slopes of the Himalayas experience snowfall
(c) During this season, the north-east trade winds blow from land to sea and hence for most parts of the country it is a dry season. Some amount of rainfall occurs on the Tamil Nadu coast from these winds as they blow there from sea to land
(d) A characteristic feature of the cold weather season over the northern plains is the inflow of cyclonic disturbances from the west and the north-west. The low pressure systems originate over the Mediterranean Sea and Western Asia and move into India along with the westerly flow. They cause the much needed winter rains over the plains and snowfall in the mountains
(e) Although, the total amount of winter rainfall locally known as 'Mahawat' is small, it is of immense importance for the cultivation of 'rabi' crops
(f) The peninsular region does not have a well defined cold season. There is hardly any noticeable change in temperature pattern during winter due to the moderating influence of the sea.

Problem 2.

(i) Name two rainiest stations.

Solution : Shillong and Mumbai.

(ii) Name two driest stations.

Solution : Leh and Jodhpur.

(iii) Name two stations with most equable climate.

Solution : Thiruvanantapuram and Mumbai.

(iv) Name two stations with most extreme climate.

Solution : Leh and Jodhpor.

(v) Name two stations most influenced by the Arabian branch of south-west monsoons.

Solution : Mumbai and Thiruvanantapuram.

(vi) Name two stations most influenced by the Bay of Bengal branch of south-west monsoons.

Solution : Shillong and Kolkata.

(vii) Name two stations influenced by both branches of the south-west monsoons.

Solution : Delhi and Nagpur.

(viii) Name two stations influenced by retreating and north-east monsoons.

Solution : Thiruvanantapuram and Chennai

(ix) Name two stations receiving winter showers from the western disturbances.

Solution : Delhi and Kolkata.

(x) Name two hottest stations in the months of

(a) February
(b) April
(c) May
(d) June

Solution :

(a) Thiruvanantapuram and Chennai.
(b) Nagpur and Chennai.
(c) Nagpur and Delhi I Jodhpur.
(d) Jodhpur and Delhi.

Problem 3. Now find out

(i) Why are Thiruvanantapuram and Shillong rainier in June than in July?

Solution : They are rainier in June as the monsoon's arrival occurs in both places in June and the initial impact of the monsoon is an intense period of heavy rain.

(ii) Why is July rainier in Mumbai than in Thiruvanantapuram?

Solution : The monsoon reaches Mumbai about 10 days after Thiruvanantapuram and so the initial impact- of the monsoon continues into the next month. After the first initial downpour the monsoon falls into a steady pattern of raining for at least a couple of hours most days. So, it reduces in Thiruvanantapuram earlier than in Mumbai.

(iii) Why are south-west monsoons less rainy in Chennai?

Solution : Chennai doesn't receive much rain during the south-west monsoon, as the south of India (the states of Tamil Nadu, Karnataka, and Kerala) gets most of its rainfall from the north-east monsoon, from October to December.

(iv) Why is Shillong rainier than Kolkata?

Solution : Shillong is in a hilly area and the hills trap the monsoon winds, so that Shillong becomes rainier than Kolkata.

(v) Why is Kolkata rainier in July than in June unlike Shillong which is rainier in June than in July?

Solution : The monsoon reaches Shillong earlier than Kolkata (refer to the 'Advancing Monsoon' map in your textbook) and the initial impact is heavier than the later showers. So, Shillong is rainier in June while Kolkata is rainier in July.

(vi) Why does Delhi receive more rain than Jodhpur?

Solution :  Jodhpur is on the edge of the Thar Desert and by the time the monsoon winds reach it, most of their moisture is finished. Delhi is more east than Jodhpur and so it receives more rainfall.

Problem 4. Now think why

(i) Thiruvanantapuram has equable climate?

Solution :  Thiruvanantapuram has equable climate because of two reasons.

(a) It is on the sea coast. The moderating influence of the sea makes the climate equable.
(b) It is near to the equator. At the equator, all the seasons have similar temperatures and so this makes the climate equable.

(ii) Chennai has more rains only after the fury of monsoon is over in most parts of the country?

Solution : Chennai receives most of its rainfall from the north-east monsoon, which gives rains mostly from October to December, and not the south-west monsoon. That is why Chennai gets most of its rainfall later than most other parts of the country.

(iii) Jodhpur has a hot desert type of climate?

Solution : Jodhpur is in the extreme western part of India and so, when the monsoon winds reach it, they have exhausted their moisture. Further it is on the edge of the Thar Desert. That is why it has a hot desert type of climate.

(iv) Leh has moderate precipitation almost throughout the year?

Solution : Leh is in the 'cold desert' called Ladakh, which is a valley in between two mountain ranges. No monsoon winds are able to reach it. That is why it has moderate precipitation almost throughout the year.

(v) While in Delhi and Jodhpur most of the rain is confined to nearly three months, in Thlruvanantapuram and Shillong it is almost nine months of the year?

Solution : Thiruvanantapuram ison the sea coast and so it receives rainfall from both the southwest and north-east monsoons, besides receiving rainfall due to local disturbances which pick up moisture from the sea. Shillong is in a hilly area and so receives rain from the monsoon as well as from local disturbances which are trapped by the hills.

(vi) In spite of these facts see carefully if there are strong evidences to conclude that the monsoons still provide a very strong framework lending overall climatic unity to the whole country.

Solution : The seasonal alteration of the wind systems and the associated weather conditions provide a rhythmic cycle of seasons. Monsoon rains are unevenly distributed and typically uncertain. The Indian landscape, plant and animal life, agriculture, the people and their festivities, all revolve around the monsoon.


(Geography) Chapter 5 Nature Vegetation & Wildlife


Problem 1. Choose the right Solution from the four alternatives given below

(i) To which one of the following types of vegetation does rubber belong to ?

(a) Tundra
(b) Tidal
(c) Himalayan
(d) Tropical Evergreen

Solution : (d)

(ii) Cinchona trees are found in the areas of rainfall more than

(a) 100 cm
(b) 50 cm
(c) 70 cm
(d) less than 50 cm

Solution : (a)

(iii) In which of the following states is the Simlipal bioReserve located ?

(a) Punjab
(b) Orissa
(c) Delhi
(d) west Bengal

Solution : (b)

(iv) Which one of the following bioreserves of India is not included in the world network of bioreserve?

(a) Manas
(b) Nilgiri
(c) Gulf of Mannar
(d) Nanda Devi

Solution : (a)

Problem 2. anwser the following questions briefly Define an ecosystem.

Solution : All the plants and animals in an area are interdependent and interrelated to each other in their physical environment, thus forming an ecosystem. Human beings are also an integral part of the ecosystem. They utilise the vegetation and wild life.

What factors are responsible for the distribution of plants and animals in India?

Solution : The factors responsible for the distribution of plants (flora) and animals (fauna) in India are

(a) Relief factors
Land
oil

(b) Climate
Temperature
Precipitation
Photoperiod (sunlight)

(iii) What is a bioreserve? Give two examples

Solution : A bioreserve is an area in which the flora and fauna of the given country is protected and there are certain researches which are done on them. It is an area containing a wildlife preserve bordered by a buffer zone in which more frequent use is permitted to the public, established as a way of integrating habitat conservation with the interests of the local community.

Examples are Rajaji in Uttarakhand and Simlipal in Orissa.

(iv) Name two animals having habitat in tropical and montane type of vegetation.

Solution : Animals found in Tropical forests are lion. tiger. pig, deer and elephant.

Animals found in Montane forests are Kashmir stag, spotted deer, wild sheep, jack rabbit, Tibetan antelope, yak, snow leopard, squirrels. shaggy horn wild ibex. bear and rare red panda, sheep and goats with thick hair.

Problem 4. Name the different types of vegetation found in India and describe the vegetation of high attitudes?

Solution : The following major types of vegetation are found in India

  • Tropical Evergreen Forests

  • Tropical Deciduous Forests

  • Tropical Thorn Forests and Scrub

  • Montane Forests

  • Mangrove Forests

  • Vegetation of High Altitude (Montane Forests)

In mountainous areas, the decrease in temperature with increasing altitude leads to a corresponding change in natural vegetation.The wet temperate type of forests are found between a height of 1000 and 2000 metres, where evergreen broad leaf trees such as oaks and chestnuts predominate.Temperate forests containing coniferous trees like pine, deodar, silver fir, spruce and cedar are found between 1500 and 3000 metres. These forests cover mostly the southern slopes of the Himalayas, places having high altitudes in southern and north east India. Temperate grasslands are common at higher elevations.At high altitudes, generally more than 3,600 metres above sea level, alpine vegetation is found. Silver fir, jumpers, pines and birches are the common trees of these forests

Problem 5. Quite a few species of plants and animals are endangered in India? Why?

Solution: Many plants and animals in India are endangered because of the greediness of human beings for their commercial value. Humans are hunting animals for their skins, horns and hooves which are In demand and give a lot of profit.Deforestation on a wide scale destroys the habitat of animals and also leads to decline of the different species of trees and plants. Ecological balance is disturbed due to deforestation, which is harmful for both flora and fauna.

Project / Activity

  • Find some trees in your neighbourhood having medicinal values.

  • Find ten occupations getting raw material from forests and wild life.

  • Write a poem or paragraph showing the importance of wild life.

  • Write the script of a street play giving the importance of tree plantation and try to enact it in your locality.

  • Plant a tree either on your birthday or one of your family member's birthday.

  • Note the growth of the tree and notice in which season it grows faster.

Solution :Some trees having medicinal values are (there are many others) Amla Plant or Indian Gooseberry This is one of the richest sources of Vitamin C. It is a medium size deciduous plant, which attains a height of 8 to 18 meters. Amla is used to make herbal products, which helps get rid of health-related questions like hair fall, haemorrhage, leucorrhoea, skin diseases and discharge of blood from uterus.

Neem Tree It plays a significant role in Ayurvedic medicine. It helps to treat chickenpox, fever, skin disease and headache

Eucalyptus It is a tall tree, with heights upto 100 metres. Oil taken out of the Eucalyptus leaves has great medicinal value. It helps in purifying blood and lowering of blood sugar level. It cures questions of asthma, bronchitis, cardiac questions and fungal Infections.

Occupations getting following raw materials from forests and wildlife

  • Carpentry

  • Rubber industry

  • Leather industry

  • Ayurvedic medicine manufacturing

  • Paper industry

  • Glue industry

  • Fruit and food production industry

  • Hunting

  • Resin extractor

  • Perfume Industry

Importance of Wildlife Wildlife comprises of the numerable varieties of wild plants, animals, fungi and microorganisms that exist on our planet Earth, rather than just cultivated plants and domesticated animals. We largely depend on this wildlife for every elementary requirement in our lite.

The food we eat, the clothes, we wear, the medicines we consume, a variety of building materials used for construction, numerous chemicals used for manufacturing our necessities, all are extracted from the wildlife existing around us. About 40,000 species of plants, animals, fungi and microscopic animals benefit us in some way or the other. The normal functioning of the biosphere depends on endless interactions amongst animals, plants, and microorganisms.

This, in turn, maintains and improves human life further. These ecological processes are vital for agriculture, forestry, fisheries and other processes that support human life. Besides, there are several biological. processes wherein wildlife plays a key role, such as pollinisation. germination, seed dispersal, soil generation, nutrient cycling, habitat maintenance, waste breakdown, and pest control.

Do it yourself.
Do it yourself.


(Geography) Chapter 6 Population


Problem 1. Choose the right Solution from the four alternatives given below

(i) Migrations change the number, distribution and composition of the population in

(a) the area of departure
(b) the area of arrival
(c) Both the area of departure and arrival
(d) None of the abov

Solution (c)

(ii) A large proportion of children in a population is a result of

(a) high birth rates
(b) high life expectancies
(c) high death rates
(d) more married couples

Solution (a)

(iii) The magnitude of population growth refers to

(a) the total population of an area
(b) the number of persons added each year
(c) the number of females per thousand males
(d) the rate at which the population increases


Solution (b)

(iv) According to the census 2001, a 'literate' person is one who

(a) can read and write his/her name
(b) can read and write any language
(c) is 7 years old and can read and write any language with understanding
(d) knows the 3 'R's (reading, writing, arithmetic)

Solution (c)

Problem 2. Solution the following Problems briefly.

(i) Why is the rate of population growth in India declining since 1981?

Solution

Since 1981, the rate of growth started declining gradually, because of popularised.

(a) Family planning measures were adopted, leading to decline in the birth rate.
(b) Awareness about advantages of small family came to be recognised.
(c) There was a growth of nuclear families occured which adopted the small family norm.
(d) Promotion of family planning programme by the government.

(ii) Discuss the major components of population growth.

Solution The major components of population growth are birth rates, death rates and migration.

The natural increase of population is the difference between birth rates and death rates.

Birth Rate Birth rate is the number of live births per thousand person in a year. It is a major component of growth, because in India birth rates have always been higher than death rates.

Death Rate Death rate is the number of deaths per thousand persons in a year. The main cause of the rate of growth of the Indian
population has been the rapid decline in death rates.

Migration Migration is the movement of people across regions and territories. Migration can be internal (within the country) or international (between the countries).

Internal migration does not change the size of the population, but influences the distribution of population within the nation.


(iii) Define age structure, death rate and birth rate.


Solution Age Structure Age structure means the number of people in different age groups in a given population.


Death Rate Death rate is the number of deaths per thousand persons In a year.
Birth Rate Birth rate is the number of live births per thousand persons in a year.


(iv) How is migration a determinant factor of population change?


Solution Migration a Determinant Factor

  • Migration is the movement' of people across regions and territories.

  • Migration can be internal (within the country) or international (between the countries).

  • Migration is a determinant factor of population change as it changes its size and composition.

  • Internal migration does not change the size of the population but influences the distribution of population within the nation.

  • In India, most migrations have been from rural to urban areas because of the 'push' factors in rural areas. These push factors are adverse conditions of

  • poverty and unemployment in the rural areas. The 'pull' factors of the city are In terms of increased employment opportunities and better living conditions.

  • These 'push' and 'pull' factors have led to increased migration from rural to urban areas and rapid rise in the urban population.

  • There has been a significant increase in the number of 'million plus cities' from 25 to 35 in just one decade i.e., 1991-2001.

Problem 3. What is the relation between occupational structure and development?

Solution The distribution of the population according to the different types of occupations is referred to as the occupational structure.

  • Occupations are generally classified as primary, secondary and tertiary.

  • Primary activities include agriculture, animal husbandry, forestry, fishing, mining and quarrying, etc.

  • Secondary activities include manufacturing industry, building and construction work, etc.

  • Tertiary activities Include transport. communications, etc.

  • The proportion of people working in different activities varies in developed and developing countries.

  • Developed nations have a high proportion of people in secondary and tertiary activities.

  • In India about 64% of the population Is engaged only in agriculture.

  • The proportion of the population dependent on secondary and tertiary sectors is about 13 and 20% respectively.

  • There has been an occupational shift in favour of secondary and tertiary sectors because of growing industrialisation and urbanisation in recent times.

Problem 4. What are the advantages of having a healthy population?

Solution Health is an important component of population composition


which affects the process of development. So we need a healthy population.

(a) The health of a person helps him/her to realise his/her potential and gives the ability to fight illness.
(b) A healthy person is an asset to the country, is more productive and helps in the progress of the country.
(c) A healthy person is able to earn more and Improve his standard of living.
(d) A healthy population makes a healthy and strong nation economically and socially.

Problem 5. What are the significant features of the National Population Policy 2000?

Solution Aims/Objectives of National Population Policy 2000

  • Imparting free and compulsory school education upto 14 years of age.

  • Reducing infant mortality rate to below 30 per 1000 live births.

  • Achieving universal immunisation of children against all vaccine preventable diseases.

  • Promoting delayed marriage for girls.

  • Making family welfare a people centered programme.

  • Protection of adolescent girls from unwanted pregnancies.

  • Protection of adolescents from Sexually Transmitted Diseases (STD) and educating them about the risks of unprotected sex.

  • Making contraceptive services 'accessible and affordable.

  • Providing food supplement and nutritional services.

  • Strengthening legal measures to prevent child marriage


(Economics)  Chapter 1 The Story of Village Palampur


Question 1. Every village in India is surveyed once is ten years during the Census and some of details are presented in the following format. Fill up the following based on information on Palampur.

(a). LOCATION:
(b). TOTAL AREA OF THE VILLAGE:
(c). LAND USE (in hectares):
(d). FACILITIES:

Question 2. Modern farming methods require more inputs which are manufactured in industry. Do you agree?

Question 3. How did the spread of electricity help farmers in Palampur?

Question 4. Is it important to increase the area under irrigation? Why?

Question 5. Construct a table on the distribution of land among the 450 families of Palampur.

Question 6. Why are the wages for farm labourers in Palampur less than minimum wages?

Question 7.
In your region, talk to two labourers. Choose either farm labourers or labourers working at construction sites. What wages do they get? Are they paid in cash or kind? Do they get work regularly? Are they in debt?

Question 8. What are the different ways of increasing production on the same piece of land? Use examples to explain.

Question 9. Describe the work of a farmer with 1 hectare of land.

Question 10. How do the medium and large farmers obtain capital for farming? How is it different from the small farmers?

Question 11. On what terms did Savita get a loan from Tajpal Singh? Would Savita's condition be different if she could get a loan from the bank at a low rate of interest?

Question 12. Talk to some old residents in your region and write a short report on the changes in irrigation and changes in production methods during the last 30 years. (Optional)


(Economics)  Chapter 2 People as Resource


Question 1. What do you understand by 'people as a resource'?

Question 2. How is human resource different from other resources like land and physical capital?

Question 3. What is the role of education in human capital formation?

Question 4. What is the role of health in human capital formation?

Question 5. What part does health play in the individual's working life?

Question 6. What are the various activities undertaken in the primary sector, secondary sector and tertiary sector?

Question 7. What is the difference between economic activities and non-economic activities?

Question 8. Why are women employed in low paid work?

Question 9. How will you explain the term unemployment?

Question 10. What is the difference between disguised unemployment and seasonal unemployment?

Question 11. Why is educated unemployed, a peculiar problem of India?

Question 12. In which field do you think India can build the maximum employment opportunity?

Question 13. Can you suggest some measures in the education system to mitigate the problem of the educated unemployed?

Question 14. Can you imagine some village which initially had no job opportunities but later came up with many?

Question 15. Which capital would you consider the best — land, labour, physical capital and human capital? Why?


(Economics)  Chapter 3 Poverty as a Challenge


Question 1. Describe how the poverty line is estimated in India.

Question 2. Do you think that present methodology of poverty estimation is appropriate?

Question 3.
Describe poverty trends in India since 1973.

Question 4.
Discuss the major reasons for poverty in India.

Question 5. Identify the social and economic groups which are most vulnerable to poverty in India.

Question 6. Give an account of interstate disparities in poverty in India.

Question 7. Describe global poverty trends.

Question 8. Describe current government strategy of poverty alleviation.

Question 9. Answer the following questions briefly

(i) What do you understand by human poverty?
(ii) Who are the poorest of the poor?
(iii) What are the main features of the National Rural Employment Guarantee Act 2005?


(Economics)  Chapter 4 Food Security in India


Question 1. How is food security ensured in India?

Question 2. Which are the people more prone to food insecurity? 3. Which states are more food insecure in India?

Question 4.
Do you believe that green revolution has made India self-sufficient in food grains? How?

Question 5. A section of people in India are still without food. Explain?

Question 6. What happens to the supply of food when there is a disaster or a calamity?

Question 7. Differentiate between seasonal hunger and chronic hunger?

Question 8.
What has our government done to provide food security to the poor? Discuss any two schemes launched by the government?

Question 9. Why is a buffer stock created by the government?

Question 10. Write notes on:

(a) Minimum support price
(b) Buffer stock
(c) Issue price
(d) Fair price shops

Question 11. What are the problems of the functioning of ration shops?

Question 12. Write a note on the role of cooperatives in providing food and related items.


(History)  Chapter 1 Events And Processes


ActivityEvents And Processes

  •  Explain why the artist has portrayed the nobleman as the spider and the peasant as the fly.

  • Fill in the blank boxes in Fig. 4 with appropriate terms from among the following: Food riots, scarcity of grain, increased number of deaths, rising food prices, weaker bodies.

  • Representatives of the Third Estate take the oath raising their arms in the direction of Bailly, the President of the Assembly, standing on a table in the centre. Do you think that during the actual event Bailly would have stood with his back to the assembled deputies? What could have been David's intention in placing Bailly (Fig.5) the way he has done?

  • 1. Identify the symbols in Box 1 which stand for liberty, equality and fraternity.

  • Explain the meaning of the painting of the Declaration of Rights of Man and Citizen (Fig. 8) by reading only the symbols.

  • Compare the political rights which the Constitution of 1791 gave to the citizens with Articles 1 and 6 of the Declaration (Source C). Are the two documents consistent? Do the two documents convey the same idea?

  • Which groups of French society would have gained from the Constitution of 1791? Which groups would have had reason to be dissatisfied? What developments does Marat (Source B) anticipate in the future?

  • Imagine the impact of the events in France on neighbouring countries such as Prussia, Austria-Hungary or Spain, all of which were absolute monarchies. How would the kings, traders, peasants, nobles or members of the clergy here have reacted to the news of what was happening in France?

  • Look carefully at the painting and identify the objects which are political symbols you saw in Box 1 (broken chain, red cap, fasces, Charter of the Declaration of Rights). The pyramid stands for equality, often represented by a triangle. Use the symbols to interpret the painting. Describe your impressions of the female figure of liberty.

  • Compare the views of Desmoulins and Robespierre. How does each one understand the use of state force? What does Robespierre mean by 'the war of liberty against tyranny'? How does Desmoulins perceive liberty? Refer once more to Source C. What did the constitutional laws on the rights of individuals lay down? Discuss your views on the subject in class.

  • Which groups of French society would have gained from the Constitution of 1791? Which groups would have had reason to be dissatisfied? What developments does Marat (Source B) anticipate in the future?

  • Imagine the impact of the events in France on neighbouring countries such as Prussia, Austria-Hungary or Spain, all of which were absolute monarchies. How would the kings, traders, peasants, nobles or members of the clergy here have reacted to the news of what was happening in France?

  • Why did people in Central Asia respond to the Russian Revolution in different ways?

  • Compare the views of Desmoulins and Robespierre. How does each one understand the use of state force? What does Robespierre mean by 'the war of liberty against tyranny'? How does Desmoulins perceive liberty? Refer once more to Source C. What did the constitutional laws on the rights of individuals lay down? Discuss your views on the subject in class.

  • Describe the persons represented in Fig. 12 – their actions, their postures, the objects they are carrying. Look carefully to see whether all of them come from the same social group. What symbols has the artist included in the image? What do they stand for? Do the actions of the women reflect traditional ideas of how women were expected to behave in public? What do you think: does the artist sympathise with the women's activities or is he critical of them? Discuss your views in the class.

  • Compare the manifesto drafted by Olympe de Gouges (Source F) with the Declaration of the Rights of Man and Citizen (Source C).

  • Record your impressions of this print (Fig. 14). Describe the objects lying on the ground. What do they symbolise? What attitude does the picture express towards non-European slaves?

  • Imagine yourself to be one of the women in Fig. 13. Formulate a response to the arguments put forward by Chaumette (Source G). Source G

  • Record your impressions of this print (Fig. 14). Describe the objects lying on the ground. What do they symbolise? What attitude does the picture express towards non-European slaves?

  • Describe the picture in your own words. What are the images that the artist has used to communicate the following ideas: greed, equality, justice, takeover by the state of the assets of the church?

Question 1. Find out more about any one of the revolutionary figures you have read about in this chapter. Write a short biography of this person.

Question2. The French Revolution saw the rise of newspapers describing the events of each day and week. Collect information and pictures on any one event and write a newspaper article. You could also conduct an imaginary interview with important personages such as Mirabeau, Olympe de Gouges or Robespierre. Work in groups of two or three. Each group could then put up their articles on a board to produce a wallpaper on the French Revolution.

Exercise Questions Events and Processes

Question 1. Describe the circumstances leading to the outbreak of revolutionary protest in France.

Question 2. Which groups of French society benefited from the revolution? Which groups were forced to relinquish power? Which sections of society would have been disappointed with the outcome of the revolution?

Question 3. Describe the legacy of the French Revolution for the peoples of the world during the nineteenth and twentieth centuries.

Question 4. Draw up a list of democratic rights we enjoy today whose origins could be traced to the French Revolution.


Question 5. Would you agree with the view that the message of universal rights was beset with contradictions? Explain.

Question 6. How would you explain the rise of Napoleon?


(History)  Chapter 2 Socialism in Europe and the Russian Revolution


Activity Socialism in Europe and
the Russian Revolution (The Age of Social Change)

  • List two differences between the capitalist and socialist ideas of private property.

  •  Imagine that a meeting has been called in your area to discuss the socialist idea of doing away with private property and introducing collective ownership. Write the speech you would make at the meeting if you are:

  • a poor labourer working in the fields

  • a medium-level landowner

  • a house owner

  • Why were there revolutionary disturbances in Russia in 1905? What were the demands of revolutionaries?

  •  The year is 1916. You are a general in the Tsar's army on the eastern front. You are writing a report for the government in Moscow. In your report suggest what you think the government should do to improve the situation.

  • Identify the symbols in Box 1 which stand for liberty, equality and fraternity.

  • Look again at Source A and Box 1.

  • List five changes in the mood of the workers.

  • Place yourself in the position of a woman who has seen both situations and write an account of what has changed.

  • Read the two views on the revolution in the countryside. Imagine yourself to be a witness to the events. Write a short account from the standpoint of:

  • an owner of an estate

  • a small peasant

  • a journalist

  • Which groups of French society would have gained from the Constitution of 1791? Which groups would have had reason to be dissatisfied? What developments does Marat (Source B) anticipate in the future?

  • Imagine the impact of the events in France on neighbouring countries such as Prussia, Austria-Hungary or Spain, all of which were absolute monarchies. How would the kings, traders, peasants, nobles or members of the clergy here have reacted to the news of what was happening in France?

  • Why did people in Central Asia respond to the Russian Revolution in different ways?

  • Compare the views of Desmoulins and Robespierre. How does each one understand the use of state force? What does Robespierre mean by 'the war of liberty against tyranny'? How does Desmoulins perceive liberty? Refer once more to Source C. What did the constitutional laws on the rights of individuals lay down? Discuss your views on the subject in class.

  • Compare the passages written by Shaukat Usmani and Rabindranath Tagore. Read them in relation to Sources C, D and E.

  • What did Indians find impressive about the USSR ?

  • What did the writers fail to notice?

  • Compare the manifesto drafted by Olympe de Gouges (Source F) with the Declaration of the Rights of Man and Citizen (Source C).

  • Imagine yourself to be one of the women in Fig. 13. Formulate a response to the arguments put forward by Chaumette (Source G).

  • Record your impressions of this print (Fig. 14). Describe the objects lying on the ground. What do they symbolise? What attitude does the picture express towards non-European slaves?

Describe the picture in your own words. What are the images that the artist has used to communicate the following ideas: greed, equality, justice, takeover by the state of the assets of the church?

Question 1. Imagine that you are a striking worker in 1905 who is being tried in court for your act of rebellion. Draft the speech you would make in your defence. Act out your speech for your class.

Question 2.
Write the headline and a short news item about the uprising of 24 October 1917 for each of the following newspapers

  • a Conservative paper in France

  • a Radical newspaper in Britain

  • a Bolshevik newspaper in Russia

Question3. Imagine that you are a middle-level wheat farmer in Russia after collectivisation. You have decided to write a letter to Stalin explaining your objections to collectivisation. What would you write about the conditions of your life? What do you think would be Stalin's response to such a farmer?

  • The French Revolution saw the rise of newspapers describing the events of each day and week. Collect information and pictures on any one event and write a newspaper article. You could also conduct an imaginary interview with important personages such as Mirabeau, Olympe de Gouges or Robespierre. Work in groups of two or three. Each group could then put up their articles on a board to produce a wallpaper on the French Revolution.

Exercise Questions Events and Processes

Question 1. What were the social, economic and political conditions in Russia before 1905?

Question 2. In what ways was the working population in Russia different from other countries in Europe, before 1917?

Question 3. Why did the Tsarist autocracy collapse in 1917?

Question4. Make two lists: one with the main events and the effects of the February Revolution and the other with the main events and effects of the October Revolution. Write a paragraph on who was involved in each, who were the leaders and what was the impact of each on Soviet history.

Question5. What were the main changes brought about by the Bolsheviks immediately after the October Revolution?

Question6. Write a few lines to show what you know about:

  •  kulaks

  •  the Duma

  •  women workers between 1900 and 1930

  •  the Liberals Activities

Question 1. Imagine that you are a striking worker in 1905 who is being tried in court for your act of rebellion. Draft the speech you would make in your defence. Act out your speech for your class.

Question 2. Write the headline and a short news item about the uprising of 24 October 1917 for each of the following newspapers

  •  a Conservative paper in France

  •  a Radical newspaper in Britain

  •  a Bolshevik newspaper in Russia

Question 3. Imagine that you are a middle-level wheat farmer in Russia after collectivisation. You have decided to write a letter to Stalin explaining your objections to collectivisation. What would you write about the conditions of your life? What do you think would be Stalin's response to such a farmer? Activities


(History) Chapter 3 Nazism and the Rise of Hitler


Activity Nazism and the Rise of Hitler

  • Read Sources A and B

  •  What do they tell you about Hitler's imperial ambition?

  •  What do you think Mahatma Gandhi would have said to Hitler about these ideas?

  •  See the next two pages and write briefly:

  •  What does citizenship mean to you? Look at Chapters I and 3 and write 200 words on how the French Revolution and Nazism defined citizenship.

  •  What did the Nuremberg Laws mean to the 'undesirables' in Nazi Germany? What other legal measures were taken against them to make them feel unwanted?

  • If you were a student sitting in one of these classes, how would you have felt towards Jews? Have you ever thought of the stereotypes of other communities that people around you believe in? How have they acquired them?

  • If you were a student sitting in one of these classes, how would you have felt towards Jews? Have you ever thought of the stereotypes of other communities that people around you believe in? How have they acquired them?

  • How would you have reacted to Hilter's ideas if you were:

  •  A Jewish woman

  • A non-Jewish German woman

  • What do you think this poster is trying to depict?

  • Look at Figs. 29 and 30 and answer the following: What do they tell us about Nazi propagarnda? How are the Nazis trying to mobilise different sections of the population?

  • Why does Erna Kranz say, 'I could only say for myself'? How do you view her opinion?

  • Write a one page history of Germany

  • as a schoolchild in Nazi Germany

  • as a Jewish survivor of a concentration camp

  • as a political opponent of the Nazi regime

  • Imagine that you are Helmuth. You have had many Jewish friends in school and do not believe that Jews are bad. Write a paragraph on what you would say to your father.

Exercise Questions Nazism and the Rise of Hitler

Question 1. Describe the problems faced by the Weimar Republic.

Question 2. Discuss why Nazism became popular in Germany by 1930.

Question 3. What are the peculiar features of Nazi thinking?

Question 4. Explain why Nazi propaganda was effective in creating a hatred for Jews.

Question 5. Explain what role women had in Nazi society. Return to Chapter 1 on the French Revolution. Write a paragraph comparing and contrasting the role of women in the two periods.

Question 6. In what ways did the Nazi state seek to establish total control over its people ?


(History) Chapter 4 Livelihoods, Economies and Societies


Activity Livelihoods, Economies and Societies

  • Each mile of railway track required between 1,760 and 2,000 sleepers. If one averagesized tree yields 3 to 5 sleepers for a 3 metre wide broad gauge track, calculate approximately how many trees would have to be cut to lay one mile of track.

  • If you were the Government of India in 1862 and responsible for supplying the railways with sleepers and fuel on such a large scale, what were the steps you would have taken?

  • An adivasi child will be able to name hundreds of species of trees and plants. How many species of trees can you name?

  • Have there been changes in forest areas where you live? Find out what these changes are and why they have happened.

  • Write a dialogue between a colonial forester and an adivasi discussing the issue of hunting in the forest.

NCERT Solutions Intext Questons

Questons 1. Discuss how the changes in forest management in the colonial period affected the following groups of people:

  • Shifting cultivators

  • Nomadic and pastoralist communities

  • Firms trading in timber/forest produce

  • Plantation owners

  • Kings/British officials engaged in shikar

Questons 2. What are the similarities between colonial management of the forests in Bastar and in Java?

Questons 3. Between 1880 and 1920, forest cover in the Indian subcontinent declined by 9.7 million hectares, from 108.6 million hectares to 98.9 million hectares. Discuss the role of the following factors in this decline:

  • Railways

  • Shipbuilding

  • Agricultural expansion

  • Commercial farming

  • Tea/Coffee plantations

  • Adivasis and other peasant users

Question 4. Why are forests affected by wars?


(History)  Chapter 5 Pastoralists in the Modern World


Activity

  • Read Sources A and B.

  • Write briefly about what they tell you about the nature of the work undertaken by men and women in pastoral households.

  • Why do you think pastoral groups often live on the edges of forests?

  • Write a comment on the closure of the forests to grazing from the standpoint of:

  • a forester

  • a pastoralist

  • Imagine you are living in the 1890s. You belong to a community of nomadic pastoralists and craftsmen. You learn that the Government has declared your community as a Criminal Tribe.

  • Describe briefly what you would have felt and done.

  • Write a petition to the local collector explaining why the Act is unjust and how it will affect your life.

  • Imagine that it is 1950 and you are a 60-year-old Raika herder living in post-Independence India. You are telling your grand-daughter about the changes which have taken place in your lifestyle after Independence. What would you say

  • Imagine that you have been asked by a famous magazine to write an article about the life and customs of the Maasai in pre-colonial Africa. Write the article, giving it an interesting title. Find out more about the some of the pastoral communities marked in Figs. 11 and 13.

NCERT Solutions Intext Questions

Question 1. Explain why nomadic tribes need to move from one place to another. What are the advantages to the environment of this continuous movement?

Question 2. Discuss why the colonial government in India brought in the following laws. In each case, explain how the law changed the lives of pastoralists:

  • Waste Land rules

  • Forest Acts

  • Criminal Tribes Act

  • Grazing Tax

Question 4. Give reasons to explain why the Maasai community lost their grazing lands.

Question 5. There are many similarities in the way in which the modern world forced changes in the lives of pastoral communities in India and East Africa. Write about any two examples of changes which were similar for Indian pastoralists and the Maasai herders.


(History)  Chapter 6 Peasants and Farmers


Activity

  • Look at the graph carefully. See how the price line moves up sharply in the 1790s and slumps dramatically after 1815. Can you explain why the line of the graph shows this pattern?

  • What happened to the women and children? Cow keeping, collection of firewood, gleaning, gathering of fruits and berries from the common lands was earlier mostly done by women and children. Can you suggest how enclosures must have affected the lives of women and children? Can you imagine how the disappearance of common lands might have changed the relationship between men, women and children within the family?

  • Read Sources C and D and answer the following.

  • What is the peasant trying to say in Source C?

  • What is John Middleton arguing?

  • Re-read from Section 1.1 to 1.4 and summarize the two sides of the argument for and against open fields. Which argument do you sympathise with?

  • On the arrows in the map indicate the commodities that flowed from one country to another.

  • Imagine that you were asked by the Emperor of China to prepare a leaflet for young people about the harmful effects of opium. Find out about the effect of opium on the human body. Design your leaflet and give it an eyecatching title

  • Imagine that you are the leader of a group of farmers protesting against having to grow opium. You have been granted a meeting with the local official of the East India Company. How would the conversation proceed? Divide the class into the two groups and act out the conversation you would have.

  • Draw a timeline from 1650 to1930 showing the significant agricultural changes which you have read about in this chapter.
    Fill in the following table with the events outlined in this chapter. Remember, there could be more than one change in a country.

NCERT Solutions Intext Questions

Question 1. Explain briefly what the open field system meant to rural people in eighteenthcentury England. Look at the system from the point of view of :

  • A rich farmer

  • A labourer

  • A peasant woman

Question 2. Explain briefly the factors which led to the enclosures in England.

Question 3. Why were threshing machines opposed by the poor in England?

Question 4. Who was Captain Swing? What did the name symbolise or represent?

Question 5. What was the impact of the westward expansion of settlers in the USA?

Question 6. What were the advantages and disadvantages of the use of mechanical harvesting machines in the USA?

Question 7. What lessons can we draw from the conversion of the countryside in the USA from a bread basket to a dust bowl?

Question 8. Write a paragraph on why the British insisted on farmers growing opium in India.

Question 9. Why were Indian farmers reluctant to grow opium?


(History)  Chapter 7 Everyday Life, Culture And Politics


Activity

  • What does the sports curriculum of a nineteenth century girls school tell us about the behaviour considered proper for girls at that time?

Question 1. Imagine a conversation between Thomas Arnold, the headmaster of Rugby School, and Mahatma Gandhi on the value of cricket in education. What would each say? Write out a conversation in the form of a dialogue.

Question 2. Find out the history of any one local sport. Ask your parents and grandparents how this game was played in their childhood. See whether it is played in the same way now. Try and think of the historical forces that might account for the changes.

NCERT Solutions Intext Questons

Question 1. Test cricket is a unique game in many ways. Discuss some of the ways in which it is different from other team games. How are the peculiarities of Test cricket shaped by its historical beginnings as a village game?

Question 2. Describe one way in which in the nineteenth century, technology brought about a change in equipment and give one example where no change in equipment took place.

Question 3. Explain why cricket became popular in India and the West Indies. Can you give reasons why it did not become popular in countries in South America?

Question 4. Give brief explanations for the following:

  • The Parsis were the first Indian community to set up a cricket club in India.

  • Mahatma Gandhi condemned the Pentangular tournament.

  • The name of the ICC was changed from the Imperial Cricket Conference to the International Cricket Conference.

  • The significance of the shift of the ICC headquarters from London to Dubai

Question 5. How have advances in technology, especially television technology, affected the development of contemporary cricket?


(History)  Chapter 8 Clothing A Social History


Activity

  • Look at Figures 2 - 5. Write 150 words on what the differences in the pictures tell us about the society and culture in France at the time of the Revolution.
     

  • Read Sources A and B. What do they tell you about the ideas of clothing in Victorian society? If you were the principal in Mary Somerville's school how would you have justified the clothing practices?
     

  • In what ways do you think these notions of weakness and dependence came to be reflected in women's clothing?
     

  • Imagine yourself to be a Muslim pleader in the Allahabad high court in the late nineteenth century. What kind of clothes would you wear? Would they be very different from what you wore at home?
     

  • These two quotations (Sources E and F), from about the same period are from two different regions of India, Kerala and Bengal. What do they tell you about the very different notions of shame regarding women's attire?
     

  • If you were a poor peasant would you have willingly taken to giving up mill-made cloth? Fig.20
     

  • Can you think of other reasons why the use of khadi could not spread among some classes, castes and regions of India?
     

  • Imagine you are the 14-year-old child of a trader. Write a paragraph on what you feel about the sumptuary laws in France.
     

  • Can you think of any expectations of proper and improper dress which exist today? Give examples of two forms of clothing which would be considered disrespectful in certain places but acceptable in others.

NCERT Solutions Intext Questions

Question 1. Explain the reasons for the changes in clothing patterns and materials in the eighteenth century.

Question 2. What were the sumptuary laws in France?

Question 3. Give any two examples of the ways in which European dress codes were different from Indian dress codes.

Question 4. In 1805, a British official, Benjamin Heyne, listed the manufactures of Bangalore which included the following:

  • Women's cloth of different musters and names

  • Coarse chintz

  • Muslins

  • Silk cloths Of this list, which kind of cloth would have definitely fallen out of use in the early 1800s and why?

Question 5. Suggest reasons why women in nineteenth century India were obliged to continue wearing traditional Indian dress even when men switched over to the more convenient Western clothing. What does this show about the position of women in society?

Question 6. Winston Churchill described Mahatma Gandhi as a 'seditious Middle Temple Lawyer' now 'posing as a half naked fakir'. What provoked such a comment and what does it tell you about the symbolic strength of Mahatma Gandhi's dress?

Question 7. Why did Mahatma Gandhi's dream of clothing the nation in khadi appeal only to some sections of Indians?


<<Go Back To Main Page